Gönderen Konu: Tübitak Lise Takım Seçme 1997 Soru 6  (Okunma sayısı 3096 defa)

Çevrimdışı Lokman Gökçe

  • Lokman Gökçe
  • Administrator
  • Geo-Maniac
  • *********
  • İleti: 3661
  • Karma: +23/-0
  • İstanbul
Tübitak Lise Takım Seçme 1997 Soru 6
« : Ağustos 08, 2013, 06:25:43 ös »
$n\ge 2$ verilmiş bir tam sayı olsun. $x_{1}^{2}+x_{2}^{2}+\ldots +x_{n}^{2}=1$ koşulunu sağlayan $x_{1},x_{2},\ldots x_{n}$ pozitif sayıları için, $$
\dfrac{x_{1}^{5}}{x_{2}+x_{3}+\ldots + x_{n}} + \dfrac{x_{2}^{5}}{x_{1}+x_{3}+\ldots + x_{n}} + \dots + \dfrac{x_{n}^{5}}{x_{1}+x_{2}+\ldots + x_{n-1}}$$ toplamının alabileceği en küçük değeri bulunuz.
« Son Düzenleme: Aralık 26, 2023, 05:07:45 ös Gönderen: Lokman Gökçe »
Uğraşınca çözebileceğim zorlukta olan soruları çözmeyi severim.

Çevrimdışı Metin Can Aydemir

  • G.O Genel Moderator
  • Geo-Maniac
  • ********
  • İleti: 1139
  • Karma: +9/-0
Ynt: Tübitak Lise Takım Seçme 1997 Soru 6
« Yanıtla #1 : Aralık 22, 2023, 01:05:29 öö »
Kesirleri $x_i$ ile genişletip, faydalı eşitsizlik (Bergström eşitsizliği) uygulayalım. $$\sum_{i=1}^{n}\frac{x_i^6}{x_i(x_1+x_2+\cdots+x_n)-x_i^2}\geq \frac{(x_1^3+x_2^3+\cdots+x_n^3)^2}{(x_1+x_2+\cdots+x_n)^2-(x_1^2+x_2^2+\cdots+x_n^2)}=\frac{(x_1^3+x_2^3+\cdots+x_n^3)^2}{(x_1+x_2+\cdots+x_n)^2-1}$$ Kuvvet ortalama eşitsizliklerinden, $$\sqrt[3]{\frac{x_1^3+x_2^3+\cdots+x_n^3}{n}}\geq \sqrt{\frac{x_1^2+x_2^2+\cdots+x_n^2}{n}}\implies x_1^3+x_2^3+\cdots+x_n^3\geq \frac{1}{\sqrt{n}}$$ elde edilir. Dolayısıyla, $$\sum_{i=1}^{n}\frac{x_i^6}{x_i(x_1+x_2+\cdots+x_n)-x_i^2}\geq\frac{1}{n}\cdot \frac{1}{(x_1+x_2+\cdots+x_n)^2-1}$$ elde edilir. Karesel-Aritmetik ortalama eşitsizliğinden $$\sqrt{\frac{x_1^2+x_2^2+\cdots+x_n^2}{n}}\geq \frac{x_1+x_2+\cdots+x_n}{n}\implies \sqrt{n}\geq x_1+x_2+\cdots+x_n$$ Bir üst eşitsizlikte kullanırsak, $$\sum_{i=1}^{n}\frac{x_i^5}{(x_1+x_2+\cdots+x_n)-x_i}\geq \frac{1}{n(n-1)}$$ elde edilir. Eşitlik durumu da $x_1=x_2=\cdots=\frac{1}{\sqrt{n}}$'dir.
« Son Düzenleme: Aralık 26, 2023, 05:07:51 ös Gönderen: Lokman Gökçe »
Gerçek hikayeler aslında söylenmeyenlerdir.

Çevrimdışı Hüseyin Yiğit EMEKÇİ

  • Geo-Maniac
  • ********
  • İleti: 602
  • Karma: +2/-0
Ynt: Tübitak Lise Takım Seçme 1997 Soru 6
« Yanıtla #2 : Aralık 22, 2023, 08:28:03 öö »
Diğer bir çözüm ise yine Titu (Faydalı Eşitsizlik veya Bergstrom Eşitsizliği) kullanarak

$$\sum_{cyc-i}{\dfrac{x_i^6}{x_i\left(x_1+x_2+\cdots+x_n\right)-x_i^2}}$$
$$\geq \dfrac{\left(x_1^2+x_2^2+\cdots+x_n^2\right)^3}{n^{3-2}\left[ \left(x_1+x_2+\cdots+x_n\right)^2-\left(x_1^2+x_2^2+\cdots+x_n^2\right)\right] }\geq \dfrac{1}{n\left (\left(x_1+x_2+\cdots+x_n\right)^2-1\right)}\geq \dfrac{1}{n\left(n-1\right)}$$

Sondaki eşitsizlik ise Karesel-Aritmetik Ortalama veya Titu ile
$$1=\dfrac{x_1^2}{1}+\dfrac{x_2^2}{1}+\cdots+\dfrac{x_n^2}{1}\overbrace{\geq}^{Titu} \dfrac{\left(x_1+x_2+\cdots+x_n\right)^2}{n}$$
olduğu sonucu $x_1+x_2+\cdots+x_n\leq \sqrt{n}$ ifadesi ile oluşturulmuştur.

« Son Düzenleme: Aralık 22, 2023, 09:34:30 öö Gönderen: geo »
''Uzman, çok dar bir alanda yapılabilecek tüm hataları yapmış kişidir.''   ~Niels Bohr

Çevrimdışı Hüseyin Yiğit EMEKÇİ

  • Geo-Maniac
  • ********
  • İleti: 602
  • Karma: +2/-0
''Uzman, çok dar bir alanda yapılabilecek tüm hataları yapmış kişidir.''   ~Niels Bohr

Çevrimdışı Lokman Gökçe

  • Lokman Gökçe
  • Administrator
  • Geo-Maniac
  • *********
  • İleti: 3661
  • Karma: +23/-0
  • İstanbul
Ynt: Tübitak Lise Takım Seçme 1997 Soru 6
« Yanıtla #4 : Aralık 26, 2023, 05:47:07 ös »
Begström eşitsizliği

$$\sum_{cyc-i}{\dfrac{x_i^6}{x_i\left(x_1+x_2+\cdots+x_n\right)-x_i^2}}\geq \dfrac{\left(x_1^2+x_2^2+\cdots+x_n^2\right)^3}{n^{3-2}\left[ \left(x_1+x_2+\cdots+x_n\right)^2-\left(x_1^2+x_2^2+\cdots+x_n^2\right)\right] } $$

biçiminde değil de

Alıntı
$$\sum_{cyc-i}{\dfrac{x_i^6}{x_i\left(x_1+x_2+\cdots+x_n\right)-x_i^2}}\geq \dfrac{\left(x_1^3+x_2^3+\cdots+x_n^3\right)^2}{\left[ \left(x_1+x_2+\cdots+x_n\right)^2-\left(x_1^2+x_2^2+\cdots+x_n^2\right)\right] } $$

biçiminde yazılabilir, diye düşünüyorum. Acaba Radon eşitsizliği mi uygulanmak istendi diye baktım. Ona da çok benzemedi. Eşitsizliğin nasıl uygulandığı ile ilgili daha fazla açıklama verilirse memnun olurum :)
Uğraşınca çözebileceğim zorlukta olan soruları çözmeyi severim.

Çevrimdışı Hüseyin Yiğit EMEKÇİ

  • Geo-Maniac
  • ********
  • İleti: 602
  • Karma: +2/-0
Ynt: Tübitak Lise Takım Seçme 1997 Soru 6
« Yanıtla #5 : Aralık 26, 2023, 08:09:14 ös »
2 şekli de doğrudur. Genelleştirilmiş Titu Eşitsizliği olarak geçmektedir ve linkte de bulunduğu üzerr Hölder Eşitsizliğinin uygulanması sonucu oluşur. İkinci çözümü eklememin sebebi direkt soruda verilen ifadeye yönelimli bir örnek teşkil etmesidir. Bağlantıdaki eşitsizlik birçok olimpiyat eşitsizliğine alternatif çözüm üretmektedir. Yakın zamanda Cebir Teoremler forumuna eklemeyi düşünüyorum daha fazla kaynak ile.
''Uzman, çok dar bir alanda yapılabilecek tüm hataları yapmış kişidir.''   ~Niels Bohr

Çevrimdışı Lokman Gökçe

  • Lokman Gökçe
  • Administrator
  • Geo-Maniac
  • *********
  • İleti: 3661
  • Karma: +23/-0
  • İstanbul
Ynt: Tübitak Lise Takım Seçme 1997 Soru 6
« Yanıtla #6 : Aralık 26, 2023, 10:58:32 ös »
Kullanılan eşitsizlik ile ilgili 2002 de bir makale (K.-C. Yang) yayınlanmış. arxiv.org daki şu makaleden https://arxiv.org/pdf/1504.05874.pdf (14. kaynakça) onun Radon eşitsizliğinin bir genellemesi olduğunu görüyoruz. Probleme Bergström eşitsizliğini uygulamadınız. İnternette gezinerek fazla zamanımızı harcamadan, çözümü rahat anlayabilmemiz için bu tür teoremlerin linkleri de verilerek yazılırsa daha faydalı olur.

Çözüm için teşekkürler.
« Son Düzenleme: Aralık 26, 2023, 11:00:14 ös Gönderen: Lokman Gökçe »
Uğraşınca çözebileceğim zorlukta olan soruları çözmeyi severim.

 


Sitemap 1 2 3 4 5 6 7 8 9 10 11 12 13 14 15 16 17 18 19 20 21 22 23 24 25 26 27 28 29 30 31 32 33 34 35 36 37 
SimplePortal 2.3.3 © 2008-2010, SimplePortal